Skip to main content
LibreTexts - Ukrayinska

1.7: Логічна еквівалентність

  • Page ID
    65145
  • \( \newcommand{\vecs}[1]{\overset { \scriptstyle \rightharpoonup} {\mathbf{#1}} } \) \( \newcommand{\vecd}[1]{\overset{-\!-\!\rightharpoonup}{\vphantom{a}\smash {#1}}} \)\(\newcommand{\id}{\mathrm{id}}\) \( \newcommand{\Span}{\mathrm{span}}\) \( \newcommand{\kernel}{\mathrm{null}\,}\) \( \newcommand{\range}{\mathrm{range}\,}\) \( \newcommand{\RealPart}{\mathrm{Re}}\) \( \newcommand{\ImaginaryPart}{\mathrm{Im}}\) \( \newcommand{\Argument}{\mathrm{Arg}}\) \( \newcommand{\norm}[1]{\| #1 \|}\) \( \newcommand{\inner}[2]{\langle #1, #2 \rangle}\) \( \newcommand{\Span}{\mathrm{span}}\) \(\newcommand{\id}{\mathrm{id}}\) \( \newcommand{\Span}{\mathrm{span}}\) \( \newcommand{\kernel}{\mathrm{null}\,}\) \( \newcommand{\range}{\mathrm{range}\,}\) \( \newcommand{\RealPart}{\mathrm{Re}}\) \( \newcommand{\ImaginaryPart}{\mathrm{Im}}\) \( \newcommand{\Argument}{\mathrm{Arg}}\) \( \newcommand{\norm}[1]{\| #1 \|}\) \( \newcommand{\inner}[2]{\langle #1, #2 \rangle}\) \( \newcommand{\Span}{\mathrm{span}}\)

    У ваших попередніх класах математики (таких як алгебра та тригонометрія) ви зустрічали багато прикладів, коли дві різні на вигляд формули виявилися рівними. Аналогічно, в Логіці може бути два різних твердження, які мають однакову цінність істини у всіх можливих ситуаціях. (Це означає, що для кожного можливого призначення true або false змінним, або обидва твердження є істинними, або обидва є помилковими.) Такі твердження, як кажуть, логічно рівнозначні.

    Позначення\(\PageIndex{1}\)

    Ми напишемо\({A} \equiv {B}\), щоб позначити,\({A}\) що логічно еквівалентно\({B}\).

    Це може зайняти багато роботи, щоб перевірити, що два твердження логічно еквівалентні. З іншого боку, щоб показати, що два твердження не є логічно еквівалентними, вам потрібно лише знайти один приклад присвоєння змінним, таким чином, що одне з тверджень є істинним, а інше - помилковим.

    Приклад\(\PageIndex{2}\)

    Якщо\(A\) істинно і\(B\) є помилковим, то\(A \lor B\) є істинним, але\(A \Rightarrow B\) помилковим. Тому твердження\(A \lor B\) і логічно\(A \Rightarrow B\) не рівнозначні.

    Вправа\(\PageIndex{3}\)

    Показати, що кожна з наступних пар речень не є логічно рівнозначними.

    1. \(A \lor B \lor \lnot C\),\((A \lor B) \& (C \Rightarrow A)\)
    2. \((P \Rightarrow Q) \lor (Q \Rightarrow P)\),\(P \lor Q\)
    3. \((X \& Y) \Rightarrow Z\),\(X \lor (Y \Rightarrow Z)\)

    Приклад\(\PageIndex{4}\)

    Покажіть, що\(\lnot(A \lor B) \equiv \lnot A \& \lnot B\).

    Рішення

    Змінні\(A\) і\(B\) можуть бути або true або false, і ми будемо оцінювати обидва твердження для всіх можливих комбінацій. Щоб було зрозуміло, що жодна з можливостей не була пропущена, ми продовжуємо систематично: для кожного значення\(A\) ми розглядаємо два можливих значення для\(B\).

    1. Припустімо\(A\), що це правда.
      1. Припустімо\(B\), що це правда. У нас є\[\lnot(A \lor B) \quad=\quad \lnot(T \lor T) \quad=\quad \lnot T \quad=\quad F\] і\[\lnot A \& \lnot B \quad=\quad \lnot T \& \lnot T \quad=\quad F \& F \quad=\quad F .\] Обидва твердження є помилковими.
      2. Припустімо\(B\), що помилково. У нас є\[\lnot(A \lor B) \quad=\quad \lnot(T \lor F) \quad=\quad \lnot T \quad=\quad F\] і\[\lnot A \& \lnot B \quad=\quad \lnot T \& \lnot F \quad=\quad T \& F \quad=\quad F .\] Обидва твердження є помилковими.
    2. Припустімо\(A\), що помилково.
      1. Припустімо\(B\), що це правда. У нас є\[\lnot(A \lor B) \quad=\quad \lnot(F \lor T) \quad=\quad \lnot T \quad=\quad F\] і\[\lnot A \& \lnot B \quad=\quad \lnot F \& \lnot T \quad=\quad T \& F \quad=\quad F .\] Обидва твердження є помилковими.
      2. Припустімо\(B\), що помилково. У нас є\[\lnot(A \lor B) \quad=\quad \lnot(F \lor F) \quad=\quad \lnot F \quad=\quad T\] і\[\lnot A \& \lnot B \quad=\quad \lnot F \& \lnot F \quad=\quad T \& T \quad=\quad T .\] Обидва твердження вірні.

    У всіх випадках або обидва твердження є істинними, або обидва є помилковими. Тому вони логічно рівнозначні.

    Ми також можемо вирішити проблему, не виконуючи стільки роботи:

    Простіше рішення

    Зауважте, що твердження\(\lnot(A \lor B)\) є істинним, якщо і тільки якщо\(A \lor B\) є помилковим, що означає, що ні\(A\) ні\(B\) є істинним. Тому,\[\text{$\lnot(A \lor B)$ is true if and only if $A$ and~$B$ are both false.}\] Крім того,\(\lnot A \& \lnot B\) є істинним тоді і тільки тоді,\(\lnot A\) коли і\(\lnot B\) є істинними, що означає, що:\[\text{$\lnot A \& \lnot B$ is true if and only if $A$ and~$B$ are both false.}\]

    Таким чином, два твердження\(\lnot(A \lor B)\) і\(\lnot A \& \lnot B\) вірні в точно тій же ситуації (а саме, коли\(A\) і обидва\(B\) є помилковими); і вони обидва є помилковими у всіх інших ситуаціях. Тому вони логічно рівнозначні.

    Вправа\(\PageIndex{5}\)

    Перевірте кожне з наступних важливих логічних еквівалентів. Для більшості з них вам не потрібно оцінювати твердження для всіх можливих значень змінних.

    1. правила заперечення:\[\begin{aligned} \lnot \lnot A \qquad &\equiv \qquad A \\ \lnot(A \& B) \qquad &\equiv \qquad \lnot A \lor \lnot B \\ \lnot(A \lor B) \qquad &\equiv \qquad \lnot A \& \lnot B \\ \lnot ( A \Rightarrow B) \qquad &\equiv \qquad A \& \lnot B \\ \lnot ( A \Leftrightarrow B) \qquad &\equiv \qquad A \Leftrightarrow \lnot B \end{aligned}\]
    2. комутативність\(\&\)\(\lor\), і\(\Leftrightarrow\):\[\begin{aligned} A \& B \qquad &\equiv \qquad B \& A \\ A \lor B \qquad &\equiv \qquad B \lor A \\ A \Leftrightarrow B \qquad &\equiv \qquad B \Leftrightarrow A \end{aligned}\]
    3. асоціативність\(\&\) і\(\lor\):\[\begin{aligned} (A \& B) \& C \qquad &\equiv \qquad A \& (B \& C) \\ (A \lor B) \lor C \qquad &\equiv \qquad A \lor (B \lor C) \end{aligned}\]

    Правила заперечення\(\&\) і\(\lor\) часто називаються законами Де Моргана, на честь британського математика Августа Де Моргана (1806—1871, http://en.Wikipedia.org/wiki/Augustus_De_Morgan).

    Правила заперечення можуть бути використані для спрощення заперечення будь-якого твердження.

    Приклад\(\PageIndex{6}\)

    Спростити\(\lnot \bigl( (A \lor B) \Rightarrow (A \& \lnot C) \bigr)\).

    Рішення

    У нас є\[\begin{aligned} \lnot \bigl( (A \lor B) \Rightarrow (A \& \lnot C) \bigr) &\equiv (A \lor B) \& \lnot (A \& \lnot C) \\& \equiv (A \lor B) \& (\lnot A \lor \lnot \lnot C) \\& \equiv (A \lor B) \& (\lnot A \lor C) . \end{aligned}\]

    Якщо\({A} \equiv {B}\), то\({A}, \ \therefore {B}\) діє відрахування. Наприклад, наведений вище приклад показує, що\[\lnot \bigl( (A \lor B) \Rightarrow (A \& \lnot C) \bigr), \ \therefore (A \lor B) \& (\lnot A \lor C)\] є дійсним відрахуванням.

    Вправа\(\PageIndex{7}\)

    Використовуйте правила заперечення, щоб спростити кожне з наступних тверджень (поки заперечення не буде застосовано ні до чого, крім змінних).

    1. \(\lnot \bigl( (A \lor B) \Rightarrow (C \& D) \bigr)\)
    2. \(\lnot \bigl( (A \Rightarrow B) \lor (C \& D) \bigr)\)
    3. \(\lnot \Bigl( A \Rightarrow \bigl( B \Rightarrow( C \Rightarrow D) \bigr) \Bigr)\)
    4. \(\lnot \Bigl( \bigl( (A \Rightarrow B) \Rightarrow C \bigr) \Rightarrow D \Bigr)\)
    5. \(\lnot \bigl( (P \lor \lnot Q) \& R \bigr)\)
    6. \(\lnot (P \& Q \& R \& S)\)
    7. \(\lnot \Bigl( \bigl( P \Rightarrow ( Q \& \lnot R) \bigr) \lor (P \& \lnot Q) \Bigr)\)

    Вправа\(\PageIndex{8}\)

    Використовуйте правила заперечення, щоб спростити заперечення кожного з цих тверджень. Висловлюйте свої відповіді англійською мовою.

    1. Якщо буде дощ, то автобус не буде вчасно.
    2. Я хворий, і я втомився.
    3. Або Папа Римський тут, або Королева і Секретар обидва тут.
    4. Якщо Том забув свій рюкзак, то Сем з'їсть або розсіл, або картоплю, і або Боб не буде обідати, або Аліса поїде в магазин.